313 exam 3

¡Supera tus tareas y exámenes ahora con Quizwiz!

A 53-year-old perimenopausal woman has visited the clinic at the urging of her sisters, who have encouraged her to investigate the possibility of beginning hormone replacement therapy (HRT). The woman tells the nurse that her sisters have touted the benefits of HRT in relieving symptoms of menopause while protecting against heart disease. How can the nurse best respond to the client?

"Actually, HRT has been found to be a threat, not a benefit, to your heart health."

A client presents to the health care clinic and reports a recent onset of a persistent cough. The client denies any shortness of breath, change in activity level, or other findings of an acute upper respiratory tract illness. What question by the nurse is most appropriate to further assess the cause for the cough?

"Are you taking any medications on a regular basis?"

A client tells a nurse that she has been experiencing intermittent episodes of numbness, tingling, pain, and burning in the fingertips, especially after being cold. What is an appropriate question for the nurse to ask the client to further assess this occurrence?

"Do you notice your fingers changing colors?"

Which statement made by a student nurse concerning how to conduct the retrograde filling (Trendelenburg) test indicates that the nurse needs further teaching?

"Observe for normal saphenous vein refill to take 30 seconds."

The nurse is providing care for a 61-year-old female smoker who is 30 kg overweight and was diagnosed with type 2 diabetes several years prior. Which of the following teaching points regarding the prevention of peripheral artery disease (PAD) is most accurate?

"Quitting smoking and keeping good control of your blood sugar levels are important."

The nurse performs an admission assessment on a 52-year-old woman admitted through the ED with a myocardial infarction. The nurse charts "Swooshing sound heard over right carotid artery." How should this documentation be corrected?

"Right carotid bruit auscultated"

A client with intermittent claudication wonders why the nurse wants to know where the client is experiencing cramping when walking. What would be the nurse's best answer?

"The area of cramping is close to the area of arterial occlusion."

A patient asks the nurse why the neck veins need to be checked when the patient is experiencing heart palpitations. What should the nurse respond to this patient?

"They help determine how much pressure is in the right atrium of the heart."

The area known as Erb's point is the third site for auscultation on the precordium. Where is it located?

*3rd left rib space*

When auscultating the heart sounds of a client, a nurse notes that the S2 is louder than the S1. How should the nurse describe S2?

*Accentuated* -Diminished -Wide split -Normal split

What term is used to describe the degree of vascular resistance to ventricular contraction?

*Afterload*

When a patient is obese or has a thick chest wall, what is difficult to palpate?

*Apical impulse* -Grade 4 murmur -Sternal angle -JVP

The nursing instructor explains to a group of students that what can shorten diastole?

*Increased HR* -BP -Filling pressure -Decreased respirations As the heart rate increases, the length of diastole is shortened. The respiratory rate, blood pressure and filling pressures do not shorten diastole.

While conducting a physical examination of the cardiovascular system, the nurse hears fine crackles on auscultation of the lungs. This finding is most likely a manifestation of which problem?

*Left-sided HF* -Hypertension -Dextrocardia -Palpitations

A nurse is assessing a client for possible dehydration. Which of the following should the nurse do?

*Observe for a decrease in venous jugular pressure* Decrease in jugular venous pressure can occur with dehydration secondary to a decrease in total blood volume, so the nurse should observe for a decrease in jugular venous pressure. Assessing the difference in the apical and radial pulses would help the nurse assess for pulse deficit. Differences in the amplitude or rate of the carotid pulse may indicate stenosis. A split S1 occurs when the left and right ventricles contract at different times (asynchronous contraction).

When evaluating the jugular venous pressure in a patient with known coronary artery disease, the nurse explains to the patient that the JVP measures the pressure in the

*Right Atrium*

The nurse is providing teaching about cardiovascular disease in a community setting. What risk factors would the nurse identify to the group as those they can modify through lifestyle choices? Select all that apply.

*Smoking* *Cholesterol* *Blood pressure* -Age -Family history

Which of the following would put the client at risk for falls? Select all that apply.

-Dizziness -Hypotension -Confusion

Before the nurse begins the physical examination of a client with congestive heart failure, the client reports having to get up at night to void frequently. Which action should the nurse take in response to the client's report?

-Ensure that the client lies flat for the examination. -*Inspect for dependent edema.* -Assess for thrills. -Palpate the carotid pulse. Dependent edema results from sodium and water reabsorption through the kidneys, leading to extracellular expansion. Increased frequency of noctouria results from the redistribution of fluid at night, forcing the client to get up to void more frequently. The client should only be told to lie flat for the physical examination if the client is hypovolemic and the neck veins need to be visualized. Palpation of the carotid pulse is useful for determining whether a murmur is systolic or diastolic. Thrills are formed by the turbulence of underlying murmurs and are associated with other cardiac conditions. P441

A client presents to the health care clinic with reports of swelling, pain, and coolness of the lower extremities. The nurse should recognize that which of these lifestyle practices are risk factors for peripheral vascular disease? Select all that apply.

-High-fat diet -Previous use of hormones -Cigarette smoking

A nurse is working with a client who demonstrates venous stasis in his legs. The nurse understands that there must be a problem with one of the mechanisms of venous function that help to propel blood back to the heart. Which of the following are included among these mechanisms? Select all that apply.

-One-way valves in the veins -Skeletal muscle contraction -Pressure gradient produced by inspiration

A client complains of chest pain. The nurse understands that chest pain can have causes other than cardiac pain, thus follows up with the client regarding the timing and quality of this pain. Which of the following would indicate cardiac pain as opposed to other types? Select all that apply.

-Worsens with activity -May occur at any time -Radiates to left shoulder and down the left arm -Has a squeezing sensation around the heart

The ABI is calculated by dividing the systolic BP at the dorsalis pedis by the systolic BP at the brachial artery. Which of the following values would be consistent with mild peripheral arterial disease?

0.85

Which reading of the ankle-brachial pressure index (ABI) should the nurse recognize as indicative of a normal healthy person?

1.00

Which reading of the ankle-brachial pressure index (ABPI) should the nurse recognize as indicative of a normal healthy person?

1.00

a normal pulse amplitude would be recorded as

2+

The nurse assesses the client's pulses to be normal. These would be documented how?

2+ Normal pulses are 2+. Absent pulses are 0. Weak pules are 1+. Increased pulses are are 3+.

A client presents with pitting edema to the left foot, which a nurse observes as a noticeably deep pit when the area is depressed and the extremity looks larger than the right. How should the nurse accurately document this amount of edema?

3+

he area known as Erb's point is the third site for auscultation on the precordium. Where is it located?

3rd left rib space

A nurse auscultates a client's carotid arteries, finding the strength of the pulse to be bounding. Which score should the nurse record?

4+ Explanation: The strength of the pulse is evaluated on a scale from 0 to 4 as follows: 0 = Absent; 1+ = Weak; 2+ = Normal; 3+ = Increased; 4+ = Bounding.

By what percent can clients reduce their risk of cardiac events the first year after quitting smoking?

50% Explanation: Nurses should ask clients who smoke about their willingness to quit at every visit. Clients who quit reduce their risk of cardiac events by 50% after the first year. Nurses can give clients choices about tools to help them quit, such as referrals to behavioral therapy, information about support groups, or medication.

The nurse is reviewing a client's cardiac output. The nurse identifies which cardiac output as being within the normal?

6 Explanation: Normal cardiac out put ranges from 5-8 L/min.

Which client is at greatest risk for the development of coronary heart disease?

65 year old male with 5 year history of diabetes

Which client is at greatest risk for development of coronary heart disease?

65-year-old male w/ 5-year history of diabetes mellitus

Blood from the lower trunk and legs drains upward into the inferior vena cava. The percentage of the body's blood volume that is contained in the veins is nearly

70%.

A nurse is receiving report from the night shift about four clients. Which client would the nurse see first?

A 64-year-old man with COPD who is short of breath and has a respiratory rate of 32 breaths/min

When auscultating the left carotid artery, the nurse notes a swishing sound. The nurse interprets this finding as suggesting which of the following?

A narrowed vessel Explanation: A swishing sound on auscultation is a bruit which is caused by turbulent blood flow through a narrowed vessel. A bruit does not indicate decreased cardiac output. Increased central venous pressure or right heart failure would be indicated by jugular venous distention

Which of the following wounds is most likely attributable to neuropathy?

A painless wound on the sole of the client's foot, which is surrounded by calloused skin

The nursing instructor is discussing assessment of the heart with students. A student states that he has a patient with a rushing vibration in the precordium that the student could feel and that it was in the area of the pulmonic valve. What should the instructor explain that the student is feeling?

A thrill

A nurse palpates the presence of an enlarged inguinal lymph node. Which area of the client's body should the nurse thoroughly examine to assess for the source of this finding?

Abdomen, noting any organ enlargement or tenderness

A nurse auscultates the heart of a client with hypertension for the past ten (10) years. With the client in the left lateral position, the nurse hears a heart sound that occurs just before S1. The nurse recognizes this sound as what pathological process?

Abnormal contraction of the ventricles due to a conduction delay

When auscultating the heart sounds of a client, a nurse notes that the S2 is louder than the S1. How should the nurse describe this heart sound? S2 is:

Accentuated Explanation: An accentuated S2 means that the S2 is louder than the S1. This occurs in conditions in which the aortic or pulmonic valve has a higher closing pressure. A diminished S2 means that the S2 is softer than the S1. This occurs in conditions in which the aortic or pulmonic valves have decreased mobility. Normal Split S2 can be heard over the second or third left intercostal space; it is usually heard best during inspiration and disappears during expiration. Wide Split S2 is an increase in the usual splitting that persists throughout the entire respiratory cycle and widens on expiration.

During the auscultation of a patient's heart sounds, the nurse asks the patient to turn onto the left side. Why is this position used during the examination?

Accentuates mitral stenosis

The nurse is caring for a client exhibiting jugular vein distention and dyspnea. The nurse auscultates an new S3 heart sound. What is the nurse's best action?

Administer prescribed diuretic. Explanation: Jugular vein distention, weight gain, dyspnea, orthopnea, PND, S3 or S4, and edema indicate excess fluid volume such as occurs with heart failure. The nurse should monitor edema, intake, and output; weigh patient daily; auscultate lung and heart sounds; administer diuretic with order; elevate head of bed for dyspnea.

When doing a shift assessment on a newly admitted client, the nurse notes lack of hair on the right lower extremity; thickened nails on the right lower digits; dry, flaky skin on the right lower extremity; and diminished tibial pulses bilaterally and absent pedal pulses. What nursing diagnosis should this client receive?

Altered tissue perfusion, arterial related to reduced blood flow

What is a long-term complication of peripheral vascular disease?

Amputation

Temporary heart pain, resolving in less than 20 minutes, aggravated by physical activity and stress is known as what?

Angina Explanation: Angina is temporary heart pain, resolving in less than 20 minutes. It can be aggravated by physical activity and stress, or there may be no triggers (unstable angina). This type of pain is not musculoskeletal, gastrointestinal, or crushing.

A client at risk for peripheral arterial disease should be screened by which of the following tests?

Ankle-brachial index

A 68-year-old retired truck driver comes to the office for evaluation of swelling in his legs. He is a smoker and has been taking medications to control his hypertension for the past 25 years. The nurse is concerned about the client's risk for peripheral vascular disease. Which of the following tests is appropriate to order to initially evaluate for this condition?

Ankle-brachial index (ABI)

When auscultating a patient's heart sounds, the nurse hears a louder S2 when listening at the 2nd intercostal space right sternal border. The nurse determines that this finding is consistent with the closure of which heart valves?

Aortic and pulmonic

When auscultating a client's heart sounds, the nurse hears a louder S2 when listening at the 2nd intercostal space right sternal border. The nurse determines that this finding is consistent with the closure of which heart valves?

Aortic and pulmonic Explanation: The closure of the aortic and pulmonic valves creates the second heart sound, which is heard louder over the 2nd intercostal space right sternal border. The closure of the tricuspid and mitral valves creates the first heart sound. The pulmonic and tricuspid valves do not close together. The mitral and aortic valves do not close together.

The nurse is palpating the apical impulse in a client with heart disease and finds that the amplitude is diffuse and increased. Which of the following conditions could be a potential cause of an increase in the amplitude of the impulse?

Aortic stenosis, with pressure overload of the left ventricle

A nurse experiences difficulty w/ palpation of the apical impulse on the precordium. What is an appropriate action by the nurse?

Ask the client to assume the left lateral position

A nurse experiences difficulty with palpation of the apical impulse on the pre cordium. What is an appropriate action by the nurse?

Ask the client to assume the left lateral position Explanation: If unable to locate the apical impulse, ask the client to turn to the left lateral position. This displaces the heart towards the left chest wall and relocates the apical impulse farther to the left. Using one finger is appropriate after locating the pulse for a more accurate palpation. Coughing will not assist in location of the apical impulse. The nurse should locate the apical impulse by palpation before auscultating heart sounds

A nurse experiences difficulty with palpation of the dorsalis pedis pulse In a client with arterial insufficiency. What is an appropriate action by the nurse based on this finding?

Assess adequacy of blood flow using Doppler device

A nurse experiences difficulty with palpation of the dorsalis pedis pulse in a client with arterial insufficiency. What is an appropriate action by the nurse based on this finding?

Assess adequacy of blood flow using a Doppler device.

A nurse assesses the peripheral vascular system of a client who is in the supine position. What further assessment should the nurse perform if unable to palpate the left popliteal pulse?

Assist the client to the prone position and palpate again

A nurse cares for a client who is postoperative cholecystectomy. Which action by the nurse is appropriate to help prevent the occurrence of venous stasis?

Assist the client to walk as soon and as often as possible.

A nurse ausculates the heart of a client w/ hypertension for the past 10 yrs. With the client in the left lateral position, nurse hears a heart sound that occurs just before S1. The nurse recognizes this sound as what pathological process?

Atrial contractions heard as vibrations against stiff-walled ventricles

The client asks the nurse what the small P wave on her ECG indicates. What would the nurse answer?

Atrial depolarization Explanation: The small P wave indicates atrial depolarization (duration up to 80 msec; PR interval 120 to 200 msec).

In which order should a nurse perform the appropriate physical assessment techniques to assess the carotid artery?

Ausculate then palpate

A client is admitted for new onset of heart failure. The nurse recognizes that which finding is the earliest sign of heart failure?

Ausculation of an S3 heart sound

Which statement describes the correct technique by a nurse for use of a stethoscope to auscultate the chest for heart sounds?

Auscultate to determine the heart rate and if the rhythm is normal Explanation: The nurse should focus on one sound at a time when auscultating the precordium. Start by determining the rate and rhythm. The examiner should stand at the client's right side to perform the assessment. The client should be lying in the supine positions with the head of the bed elevated at 30 degrees. The diaphragm of the stethoscope is used to listen for the high pitched should of normal heart sounds.

hich statement describes the correct technique by a nurse for use of a stethoscope to auscultate the chest for heart sounds?

Auscultate to determine the heart rate and if the rhythm is normal Explanation: The nurse should focus on one sound at a time when auscultating the precordium. Start by determining the rate and rhythm. The examiner should stand at the client's right side to perform the assessment. The client should be lying in the supine positions with the head of the bed elevated at 30 degrees. The diaphragm of the stethoscope is used to listen for the high pitched should of normal heart sounds.

A client with peripheral vascular disease is discharged from the health care facility. Which risk-reduction teaching tip should the nurse discuss during discharge teaching?

Avoid smoking

When assessing temperature of the skin, which portion of the hand should the examiner use?

Backs of fingers

A nurse is interviewing a client who complains of dyspnea of sudden onset. Based on this finding, the nurse should suspect which of the following causes?

Bacterial Infection

A client presents to the emergency department complaining of chest pain. The nurse conducts a pain assessment and discovers the client's chest pain has lasted more than 20 minutes and is accompanied by nausea and diaphoresis. The nurse should prepare for which treatment?

Balloon angioplasty Explanation: The client's symptoms are consistent with a myocardial infarction. Treatment includes nitroglycerin, bedrest to decreased oxygen consumption, thrombolytics, or angioplasty. Open heart surgery may be indicated, depending on the results of the angioplasty. Pain medication may be prescribed along with other treatments for myocardial infarction; but morphine alone, with only observation is not likely.

A nurse cares for a client admitted after falling off a ladder onto a concrete floor. The client is not arousable and pupils are fixed and dilated. When performing a respiratory assessment, the nurse recognizes which breathing pattern as normal for clients with brain damage?

Biot's

A client is experiencing decreased cardiac output. Which vital sign is priority for the nurse to monitor frequently?

Blood pressure Explanation: With decreased cardiac output, the heart pumps inadequate blood to meet the body's metabolic demands. The blood pressure is most important to assess frequently.

Which pulse is located at approximately the inner third of the antecubital fossa when the palm is held upward?

Brachial

A nurse auscultates a client's heart sounds and obtains a rate of 56 beats per minute. How should this rate be documented by the nurse?

Bradycardia Explanation: The proper documentation of this rate is bradycardia, a rate less than 60 beats per minute. The normal adult heart rate is 60 to 100 beats per minute. Tachycardia is a heart rate above 100 beats per minute. This heart rate is decreased, but this is not a proper documentation term.

A nurse performs a respiratory assessment on a client and notes the respiratory rate to be 8 breaths per minute. The nurse knows the proper term for this rate is what?

Bradypnea

The client is experiencing septic shock. What assessment finding would the nurse expect to find?

Capillary refill greater than 2 seconds

The nurse assesses a client's neck as shown. What is the nurse assessing?

Carotid artery

Which of the following anterior neck structures is found in the depression between the trachea and the sternomastoid muscle?

Carotid artery

A nurse is assessing a client for the presence of stenosis in the carotid arteries. Which of the following should the nurse do?

Check for pulse inequality between right and left carotid arteries Explanation: The nurse should check for pulse inequality between the right and left carotid arteries, because differences in the amplitude or rate of the carotid pulse may indicate stenosis. Pulse deficit is detected by assessing the difference in the apical and radial pulses. A split S1 occurs when the left and right ventricles contract at different times (asynchronous contraction). Decrease in jugular venous pressure can occur with dehydration secondary to a decrease in total blood volume.

A client with a right subclavian central line develops fever of 101.0 degrees Fahrenheit. What is the nurse's best action?

Check the insertion site for redness.

Upon inspection of a client's chest, a nurse observes an increase in the anterior posterior diameter. The nurse recognizes this as a finding in which disease process?

Chronic Obstructive Pulmonary Disease

A client presents to the health care clinic with reports of swelling, pain, and coolness of the lower extremities. The nurse should recognize that which of these lifestyle practices are risk factors for peripheral vascular disease? Select all th

Cigarette smoking Previous use of hormones High-fat diet

A nurse is unable to palpate the apical impulse on a client. Which assessment data in the client's history should the nurse recognize as the reason for this finding?

Client has an increased chest diameter

What is responsible for the inspiratory splitting of S2?

Closure of aortic then pulmonic valves Explanation: During inspiration, the closures of the aortic valve and pulmonic valves separate slightly, and this may be heard as two audible components instead of as a single sound. Current explanations of inspiratory splitting include increased capacitance in the pulmonary vascular bed during inspiration, which prolongs ejection of blood from the right ventricle, delaying closure of the pulmonic valve. Because the pulmonic component is soft, the examiner may not hear it away from the left second intercostal space. Because it is a low-pitched sound, the examiner may not hear it without use of the bell of the stethoscope. It is generally easy to hear in school-aged children, and it is easy to notice the respiratory variation of the splitting

Which of the following events occurs at the start of diastole?

Closure of the aortic valve Explanation: At the beginning of diastole, the valves that allow blood to exit the heart close. It is thought that the closure of the aortic valve produces the second heart sound (S2). Closure of the mitral valve is thought to produce the first heart sound (S1).

The nurse is analyzing the data from the assessment of a client's heart and neck vessels. The client's first heart sound corresponds with what event in the cardiac cycle?

Closure of the atrioventricular valves Explanation: The first heart sound is the result of closure of the atrioventricular valves. The second heart sound is the result of closure of the semilunar valves. Ventricular contraction is isometric when all four valves are closed during systole. Diastole occurs when the AV valves are open and the ventricles are relaxed.

The nurse is assessing a client's first heart sound. The nurse interprets this finding as indicating which heart action?

Closure of the atrioventricular valves Explanation: The first heart sound is the result of closure of the atrioventricular valves. The second heart sound is the result of closure of the semilunar valves. Ventricular contraction is isometric when all four valves are closed during systole. Diastole occurs when the AV valves are open and the ventricles are relaxed.

A nurse understands that the cardiac event that cycles the beginning of systole is what?

Closure of the mitral & tricuspid valves

A nurse understands that the cardiac event that signals the beginning of systole and produces the first heart sound is what?

Closure of the mitral and tricuspid valves Explanation: The beginning of systole occurs when the pressure in the ventricles exceeds the pressure in the atrium, causing the mitral and tricuspid valves to close. This closure produces the first heart sound (S1). The ventricles contract and empty of the blood volume, which causes the pressure to drop and the aortic and pulmonic valves close. This produces the second heart sound (S2). Relaxation and contraction of the ventricles do not produce heart sounds.

A nurse recognizes that the second heart sound, S2, is produced by which cardiac action?

Closure of the semilunar valves Explanation: Closure of the semilunar valves, which are the aortic and pulmonic valves, causes the second heart sound, S2. The closure of these valves signals the end of systole. Isometric contraction occurs when all valves are closed, which occurs just before systole, in which no sound is produced. Closure of the AV valves produces the S1 heart sound, which is the beginning of systole. Ventricular contraction is the occurrence of systole, which produces not sound but causes ejection of blood from the ventricles.

During the admission assessment, the nurse identifies the client has a history of Raynaud's. What assessment finding would the nurse expect to find?

Cold fingers and hands

Which of the following assessment findings is most congruent with chronic arterial insufficiency?

Cool foot temperature and ulceration on the client's great toe

A nurse palpates a client's hands and fingers. Which of the following findings would be consistent with arterial insufficiency?

Cool skin

The nurse understands that when the sympathetic nervous system is stimulated what occurs? Select all that apply.

Correct response: • Increased cardiac output • Increased blood pressure • Increased heart rate Explanation: When the sympathetic nervous system is stimulated, epinephrine and norepinephrine are released which causes an increased heart rate and cardiac output and and increase in the blood pressure. (less) Reference: Weber, J., and Kelley, J. Health Assessment in Nursing, 5th ed., Philadelphia: Wolters Kluwer Health, 2014, Chapter 21: Heart and Neck Vessels, pg. 422.

When auscultating a client's heart sounds the nurse detects a murmur that is initially loud and then gets softer. The nurse determines the pattern of this murmur to be which of the following?

Crescendo-decrescendo Explanation: A crescendo-descrescendo murmur is one that grows louder and then grows softer. A crescendo murmur is one that grows louder while a decrescendo murmur is one that grows softer. A plateau murmur stays the same throughout.

A nurse recognizes which finding as an indication of an ulcer due to arterial insufficiency?

Deep ulcers that often involve joint space

On questioning a client with peripheral edema during an interview, the nurse learns that the client has a sedentary job and drinks little water throughout the day. What underlying condition is the client most likely to have, based on these findings?

Deep vein thrombosis

Which of the following muscles is primarily responsible for thoracic cavity enlargement?

Diaphragm

The nurse is assessing a client with mitral insufficiency. Which characteristic of the first heart sound should the nurse expect to hear?

Diminished Explanation: A client with mitral insufficiency would most likely exhibit a diminished S1 sound. A split S1 sound would be heard with conduction delays and ventricular ectopy. An accentuated S1 sound would be heard in hyperkinetic states and mitral stenosis. Varying S1 sound would be heard with atrial fibrillation.

Which characteristic of the apical pulse should a nurse expect to find in the client diagnosed w/ left ventrical hypertrophy?

Displaced

A nurse assesses capillary refill time in a client and finds it to be less than 2 seconds. What is an appropriate action by the nurse?

Document the finding as normal.

In assessing a client, a nurse palpates her epitrochlear lymph nodes and notes that the client may have an infection in the hand or forearm. The nurse understands that which of the following are functions of the lymphatic system? Select all that apply.

Drains excess fluid and plasma proteins from tissues and returns them to the venous system Traps and destroys microorganisms and foreign materials filtered from lymph Absorbs fats from the small intestine into the bloodstream

The nurse is caring for a patient who has an elevated cholesterol level. To reduce the mean total blood cholesterol and LDL cholesterol levels, what would be important to teach this patient?

Eat low-fat, low-cholesterol meals

A nurse observes a decrease in hair on the lower extremities of an elderly client. What is an appropriate action by the nurse in regards to this finding?

Elevate the legs and observe for the onset of pallor.

The nurse is caring for a client with venous ulcers on both legs. The client is complaining of pain. What is the nurse's best action?

Elevate the legs on pillows.

The physician is preparing to insert a radial arterial line. What test must be performed prior to insertion?

Elevate the legs on pillows.

A nurse performs the Allen test to evaluate the patency of the radial and ulnar arteries for a client who is to undergo a radial artery puncture. What precaution should the nurse take to prevent a false-positive test?

Ensure that the client's hand is not opened in exaggerated extension

A patient has developed an infection of the right forearm. The nurse will focus the assessment of the patient's lymphatic system on which area?

Epitrochlear

The client has been admitted through the emergency department with chronic bronchitis, has elevated CO2 levels, and has been placed on O2. What priority assessment would the nurse include?

Evaluate changes in respiratory pattern and rate

Which of the following is an essential topic when discussing risk factors for peripheral arterial disease with a client?

Extent of tobacco use and exposure

To function adequately, the nurse knows that the heart valves need to open simultaneously.

False

To function adequately, the nurse knows that the heart valves need to open simultaneously. (T/F)

False

Walking contracts the calf muscles and forces blood away from the heart.

False

Which of the following arteries can be palpated below the inguinal ligament between the anterior superior iliac spine and the symphysis pubis?

Femoral artery

After teaching a group of students about the great vessels, the instructor determines that the students need additional teaching when they identify which of the following as a great vessel?

Femoral artery Explanation: The large veins and arteries leading directly to and away from the heart are the great vessels and include the superior and inferior vena cava, the pulmonary artery and vein, and the aorta. The femoral artery is a distracter for the question.

The nurse is preparing to assess a client's apical impulse. The nurse should palpate at which location?

Fifth intercostal space, left midclavicular line Explanation: The apical impulse is palpated at the fourth or fifth intercostal space at the left midclavicular line.

A nurse receives an order to perform a compression test to assess the competence of the valves in a client's varicose veins. Which action by the nurse demonstrates the correct way to perform this test?

Firmly compress the lower portion of the varicose vein

A patient with dehydration or volume depletion has barely visible neck veins, even when lying flat. These are described as what?

Flat neck veins

What would the nurse expect to hear when auscultating the lungs of a client with pleuritis?

Friction rub

A grandmother brings her 13-year-old grandson for evaluation. She noticed last week when he took off his shirt that his breastbone seemed collapsed. He seems embarrassed and says that it has been that way for awhile. He states he has no symptoms from it and that he just tries not to take off his shirt in front of anyone. He denies any shortness of breath, chest pain, or lightheadedness on exertion. His past medical history is unremarkable. He is in sixth grade and just moved in with his grandmother after his father was transferred for a work contract. His mother died several years ago in a car accident. He states that he does not smoke and has never touched alcohol. Examination shows a teenage boy appearing his stated age. Visual examination of his chest reveals that the lower portion of the sternum is depressed. Auscultation of the lungs and heart is unremarkable. What disorder of the thorax best describes these findings?

Funnel Chest

The nurse is reviewing the client's health history and notes he has pectus excavatum. The nurse would assess the client for what?

Funnel Chest

The nurse is participating in a health fair and performing cholesterol screens. One person has hypercholesterolemia. She is concerned about her risk for developing heart disease. Which of the following factors is used to estimate the 10-year risk of developing coronary heart disease?

Gender

A nurse auscultates a very loud murmur that occurs throughout systole and can be heard with the stethoscope partly off the chest. How should the nurse grade this murmur?

Grade 5 Explanation: A very loud murmur that can be heard with the stethoscope partly off the chest is graded as Grade 5. A Grade 1 is very faint and a Grade 6 can be heard with the stethoscope entirely off the chest. A Grade 2 is quiet but heard immediately on placing the stethoscope on the chest.

A nurse auscultates a murmur that occurs throughout systole and can be heard with the stethoscope partly off the chest. How should the nurse grade this murmur?

Grade V/VI

Adventitious sounds are heard when auscultating a client's lungs. Which of the following would the nurse do first?

Have the client cough and then listen again

How does the nurse differentiate a pleural friction rub from a pericardial friction rub?

Have the client hold his or her breath; if the rub persists, it is pericardial Pericardial friction rubs can be differentiated from pleural friction rubs by having the client hold the breath. If present without breathing, the rub is pericardial. Turning the client to the right side and auscultating either the base of the heart or the upper back do not differentiate between pericardial and pleural friction rubs.

The finding of a fourth heart sound (S4) is considered benign under which of the following conditions?

Healthy older adulthood

A client has sought care with complaints of increasing swelling in her feet and ankles, and the nurse's assessment confirms the presence of bilateral edema. The nurse's subsequent assessments should focus on the signs and symptoms of what health problem?

Heart failure Explanation: Edema in both lower extremities at night is seen in heart failure due to a reduction of blood flow out of the heart causing blood returning to the heart to back up in the organs and dependent areas of the body. Edema is not associated with MI, heart block, or atherosclerosis.

The nurse's assessment of a client reveals jugular venous distention. The nurse should conduct further assessments related to what health problem?

Heart failure Explanation: Jugular venous distention (JVD) is associated with heart failure, tricuspid regurgitation, and fluid volume overload. The neck veins appear full, and the level of pulsation may be have elevated jugular venous pressure greater than 3 cm (about 1 1/4 in.) above the sternal angle. About 75% of patients with elevated JVD have heart failure.

A nurse is preparing a teaching plan for a client newly diagnosed with peripheral arterial disease. To address the most modifiable risk factors, what risk factors would the nurse include? (Mark all that apply.)

High-fat diet Smoking Activity level

A client presents with lymphedema in one arm, with nonpitting edema. Which of the following should the nurse assess for, based on this finding?

History of breast surgery

A 77-year-old retired nurse has an ulcer on a lower extremity. All the following diseases are responsible for causing ulcers in the lower extremities except for:

Hypertension

A nurse is preparing a health education session for a local community group. When addressing the relationship between coronary artery disease (CAD) and culture, which information would the nurse include?

Hypertension is more prevalent in African Americans than among Caucasians. Explanation: Ethnicity plays a role in developing coronary heart disease. African Americans, Mexican Americans, American Indians, native Hawaiians, and some Asian Americans have a higher risk of heart disease thought to be due to more severe hypertension and higher rates of obesity and diabetes in these populations.

The nurse notes the client has weak pulses bilaterally. The nurse understands that this could indicate the client is experiencing what?

Hypervolemia

The nurse notes that a client's lower left leg swelling is alleviated when the extremity is elevated. Which stage of lymphedema is this client experiencing?

I

The nurse is assessing a client who has a complex cardiac history. The nurse has asked the client to lean forward while in a sitting position. This position will allow the nurse to do which of the following?

Identify heart sounds that may be inaudible in other positions. Explanation: Otherwise undetectable heart sounds may be revealed with alternative positioning. These positions are not adopted primarily to relieve shortness of breath or to assess mobility.

The nurse is planning care for a patient recovering from orthopedic surgery. Interventions should be included to address which contributing factor to deep vein thrombosis development?

Immobility

A client presents to the health care clinic with a 3-week history of pain and swelling of the right foot. A nurse inspects the foot and observes swelling and a large ulcer on the heel. The client reports the right heel is very painful and he has trouble walking. Which nursing diagnosis should the nurse confirm from these data?

Impaired Skin Integrity

A client presents to the health care clinic wit ha 3-week history of pain and swelling of the right foot. A nurse inspects the foot and observes swelling and a large ulcer on the heel. The client reports the right heel is very painful and he has trouble walking. Which nursing diagnosis should the nurse confirm from these data?

Impaired skin integrity

Where is the point of maximal impulse (PMI) normally located?

In the left 5th intercostal space 7 to 9 cm lateral to the sternum Explanation: The PMI is usually located in the left 5th intercostal space, 7 to 9 cm lateral to the sternal border. If it is located more laterally, it usually represents cardiac enlargement. Its size should not be greater than the size of a quarter or around 1 to 2.5 cm. Left ventricular enlargement should be suspected if it is larger. The PMI is often the best place to listen for mitral valve murmurs as well as S3 and S4. The PMI is often difficult to feel in normal clients.

A nurse is having trouble finding the apical pulse on an obese person. What is the most likely reason for this?

Increased distance from the apex of the heart to the pre cordium Explanation: In addition, it may be difficult to palpate the apical impulse in clients who are obese or barrel chested because these conditions increase the distance from the apex of the heart to the pre cordium. The other answers are not likely reasons for the nurse's inability to find the apical pulse.

A client presents to the health care facility with sudden onset of shortness of breath, inability to lie flat, and a deep, wet cough. A nurse observes a respiratory rate of 18 breaths per minute, use of accessory muscles to breathe, and inability to cough up secretions. Which nursing diagnosis can be confirmed with this data?

Ineffective Airway Clearance

A client is admitted to the health care facility with reports of chest pain, elevated blood pressure and shortness of breath with activity. The nurse palpates the carotid arteries as 1+ bilaterally and a weak radial pulse. A Grade III/VI systolic murmur is auscultated. Which nursing diagnosis can the nurse confirm based on this data?

Ineffective Tissue Perfusion

A nurse cares for a client who suffered a myocardial infarction 2 days ago. A high-pitched, scratchy, scraped sound is heard that increases w/ exhalation & when the client leans forward. The nurse recognizes this sound as a result of what processes occurring within the pericardium?

Inflammation of the pericardial sac

A nurse auscultates a client's lungs and hears fine crackles. What is an appropriate action by the nurse?

Instruct the client to cough forcefully

A 57-year-old maintenance worker comes to the office for evaluation of pain in his legs. He is a two-pack per day smoker since the age of 16, but he is otherwise healthy. The nurse is concerned that the client may have peripheral vascular disease. Which of the following is part of common or concerning symptoms for the peripheral vascular system?

Intermittent claudication

The nurse is assessing an older adult. The client states that she feels a constant, sharp pain only when walking. The nurse suspects the client is experiencing what?

Intermittent claudication

Which is true of a third heart sound (S3)?

It is caused by rapid deceleration of blood against the ventricular wall. Explanation: The S3 gallop is caused by rapid deceleration of blood against the ventricular wall. S4 is heard with atrial contraction and is absent in atrial fibrillation for this reason. It usually indicates a stiff or thickened left ventricle as in hypertension or left ventricular hypertrophy.

To assess the function of the right side of the heart, a nurse should perform which part of the heart & neck vessel assessment?

Jugular venous pulse

When performing a physical examination for a client with scoliosis, which physical characteristic should the nurse expect to find during the assessment?

Lateral deviation of the spinous processes

A nurse suspects that a client may have a pericardial friction rub. To ensure that the nurse hears this, the nurse would place the client in which position?

Leaning forward while in a sitting position Explanation: For best results, the nurse would use the diaphragm of the stethoscope and have the client sit up, lean forward, exhale, and hold his or her breath. The left lateral position may be used to hear an S3 or S4 heart sound or a murmur of mitral stenosis that was not detected in the supine position.

A nurse performs the Trendelenburg test for a client with varicose veins. Which action should the nurse take when performing this test?

Legs should be elevated for 15 seconds

A nurse has just inspected a standing client's legs for varicosities. The nurse would now like to assess for suspected phlebitis. Which of the following should the nurse do next?

Lightly palpate the client's leg veins for tenderness

What mechanisms produce edema? (Select all that apply.)

Low plasma protein levels Capillary leak syndrome Increased capillary blood pressure

A nurse palpates the presence of an enlarged epitrochlear lymph node. Which area of the client's body should the nurse thoroughly examine to assess for the source of this finding?

Lower arm and hand for erythema and swelling

A nurse recognizes that what change is considered normal in the lymphatic system of the elderly?

Lymph nodes are smaller and fewer in number.

The client has a history of breast cancer with reconstructive surgery. The nurse should assess the client for what potential complication?

Lymphedema

The lymphatic system functions to? (Mark all that apply.)

Maintain protein balance Fight infection Maintain fluid balance

Before beginning the assessment of the peripheral vascular system, a nurse should take what action to best facilitate the exam and ensure accurate results?

Make sure the temperature in the room is comfortable

A nurse inspects the lower extremities of a client and notices that the legs appear asymmetric. What should the nurse do first in regards to this finding?

Measure the diameter of the calves

A nurse expects to find which abnormal heart sound in a client diagnosed with mitral valve prolapse?

Midsystolic click Explanation: The nurse would expect to find a midsystolic click on auscultation in the client diagnosed with mitral valve prolapse. A ventricular gallop is the third heart sound and is associated with decreased myocardial contractility, myocardial failure, congestive heart failure, and volume overload of the ventricle. A venous hum is a benign sound caused by turbulence of blood in the jugular veins. An opening snap is heard with the opening of a stenotic or stiff mitral valve.

Variations in the presentation of S1 are d/t alterations in which heart valve?

Mitral

A 25-year-old optical technician comes to the clinic for evaluation of fatigue. As part of the physical examination, the nurse listens to her heart and hears a murmur only at the cardiac apex. Which valve is most likely to be involved based on the location of the murmur?

Mitral Explanation: Mitral valve sounds are usually heard best at and around the cardiac apex.

The nurse is performing a cardiac examination of a client with shortness of breath and palpitations. The nurse listens to the heart with the client sitting upright, then has him change to a supine position, and finally has him turn onto his left side in the left lateral decubitus position. Which of the following valvular defects is best heard in this position?

Mitral Explanation: The left lateral decubitus position brings the left ventricle closer to the chest wall, allowing mitral valve murmurs to be better heard. If the examiner does not listen in a quiet room to the heart in this position with both the diaphragm and bell, it is possible to miss significant murmurs such as mitral stenosis

Variations in the presentation of S1 are due to alterations in which heart valve?

Mitral Explanation: The sound of S1 is produced at the onset of systole, which is the closure of the mitral and tricuspid valves. The variations in the intensity of S1 are due to the position of the mitral valve at the start of systole and can cause the sound to be accentuated, diminished, or variable. The tricuspid valve is involved when there is a split S1, which causes the ventricles to contract at different times. The aortic and pulmonic valve closures produce the sound of S2.

A nurse auscultates a client's heart sounds and notes an accentuated first heart sound. The nurse would suspect which of the following?

Mitral stenosis Explanation: An accentuated S1 sound is louder than an S2 sound. This occurs when the mitral valve is wide open and closes quickly such as in hyperkinetic states (e.g., fever, anemia, hyperthyroidism) or mitral stenosis. A heart murmur reflects turbulent blood flow.

A group of students is reviewing the structures of the heart, noting that the thickest layer of the heart is made up of contractile muscle cells. The students are correct in identifying this layer as which of the following?

Myocardium Explanation: The myocardium is the thickest layer of the heart and is made up of contractile cardiac muscle cells. The epicardium is the serous membrane that covers the outer surface of the heart; the endocardium is a thin layer of endothelial tissue that forms the innermost layer of the heart. The pericardium is a tough indistensible loose-fitting fibroserous sac that attaches to the great vessels and thereby surrounds the heart.

A nurse performs an initial health history on a client admitted for new onset of chest pain. WHich data are considered subjective for the cardiovascular system?

No current medications or treatments

If palpable, superficial inguinal nodes are expected to be:

Nontender, mobile, and 1 cm in diameter

A nurse assists the client to perform the position change test for arterial insufficiency. While the client is dangling the legs, the nurse observes a return of color to the feet in 8 seconds. How should the nurse document the finding for this test?

Normal (Less than 10 seconds)

When assessing the lymph system of an adult client, the nurse notes that the epitrochlear nodes are nonpalpable. What does this indicate?

Normal finding

A hospitalized post-operative client exhibits edema, pain, erythema, and warmth in the right calf area. What is the nurse's best action?

Notify the healthcare provider.

A nurse observes a client sitting in the tripod position. What is an appropriate action by the nurse in response to this observation?

Observe for the use of accessory muscles

An elderly client reports a feeling of dyspnea with normal activities of daily living. What is an appropriate action by the nurse?

Observe the client' respiratory rate and pattern

A patient complains of difficulty sleeping. He states he has to sit up with the help of several pillows and cannot breathe when lying flat. This patient has a condition known as what?

Orthopnea

which is dyspnea that occurs when lying down and improves when sitting up

Orthopnea

A nurse recognizes that a common complication of vascular surgery may manifest as which assessment finding?

Pain in the calf muscles

A client is admitted with leg ulcers to the health care facility. During the collection of objective data, which assessment finding should indicate to the nurse that the client's leg ulcers are due to arterial insufficiency?

Pallor of foot occurs with elevation

The nurse is conducting a workshop on the measurement of jugular venous pulsation. As part of instruction, the nurse tells the students to make sure that they can distinguish between the jugular venous pulsation and carotid pulse. Which of the following characteristics is typical of the carotid pulse?

Palpable Explanation: The carotid pulse is palpable; the jugular venous pulsation is rarely palpable. The carotid upstroke is normally brisk, but may be delayed and decreased as in aortic stenosis or bounding as in aortic insufficiency.

A nurse palpates a weak left artery on a client. What should the nurse do next?

Palpate both radial arteries for symmetry

A nurse palpates a weak left radial artery on a client. What should the nurse do next?

Palpate both radial arteries for symmetry

The nurse is preparing to assess a client's carotid arteries. Which of the following would be most appropriate?

Palpate each artery individually to compare Explanation: When assessing a client's carotid arteries, the nurse should palpate each artery individually because bilateral palpation could result in reduced cerebral blood flow. Auscultation should be done before palpation because palpation may increase or slow the heart rate, changing the strength of the carotid pulse heard. The nurse should use the bell of the stethoscope to auscultate the arteries and have the client hold his or her breath for a moment so breath sounds do not conceal any vascular sounds.

A nurse experiences difficulty differentiating S1 from S2 when auscultating a client's heart sounds. What is an appropriate aciton by nurse?

Palpate the carotid pulse simultaneously

A nurse experiences difficulty differentiating S1 from S2 when auscultating a client's heart sounds. What is an appropriate action by the nurse?

Palpate the carotid pulse simultaneously

A nurse is having difficulty identifying a client's heart sounds, specifically S1 and S2. Which of the following would be most appropriate for the nurse to do?

Palpate the carotid pulse while auscultating the heart. Explanation: If a nurse is having difficulty differentiating S1 from S2, the nurse should palpate the carotid pulse while auscultating the heart. The harsh sound that occurs with the carotid pulse is the S1 sound. The nurse should use the diaphragm of the stethoscope to auscultate S1 and S2 heart sounds. A pulse deficit is determined if the heart rhythm is found to be irregular. Palpating the apical impulse would not provide any help in differentiating S1 and S2 sounds.

The nurse is assessing the carotid arteries of a client with a history of heart disease. What action should the nurse perform during this assessment?

Palpate the client's carotid arteries gently if an occlusion is audible. Explanation: If you detect occlusion during auscultation, palpate very lightly to avoid blocking circulation or triggering vagal stimulation and bradycardia, hypotension, or even cardiac arrest. Palpation should be performed alternately to avoid cerebral ischemia, and the client should briefly hold the breath during auscultation. Auscultation should precede palpation

A nurse cares for a client w/ acute pericarditis. The nurse should monitor the client for onset of which clinical manifestation of cardiac tamponade?

Paradoxical pulse

A nurse detects a bruit on ausculation of carotid arteries. What percaution should nurs take during remainder of physical assessment of carotid arteries?

Perform palpation lightly

A nurse detects a bruit on auscultation of the carotid arteries. What precaution should the nurse take during the remainder of the physical assessment of the carotid arteries?

Perform palpation lightly Explanation: The presence of a bruit indicates occlusion of the carotid artery & pressing on the artery could compromise blood flow to the brain. On detecting the bruit of the carotid artery, the nurse should palpate very lightly to avoid blocking circulation or triggering vagal stimulation and bradycardia, hypotension, or even cardiac arrest. In particular, avoid area of the carotid sinus. Making the client sit in an upright position and preventing frequent position changes is not necessary. Performing only auscultation may not give complete information.

While performing an admission assessment, the nurse auscultates a high-pitched, scratching, and grating sound at the left lower sternal border. The nurse should know that this would be documented as what type of sound?

Pericardial friction rub

a ______________________-- is the most important physical sign of acute pericarditis.

Pericardial friction rub

What is the most important physical sign of acute pericarditis?

Pericardial friction rub Explanation: A pericardial friction rub is the most important physical sign of acute pericarditis. It may have up to three components during the cardiac cycle and is high pitched, scratching, and grating. It can best be heard with the diaphragm of the stethoscope at the left lower sternal border. The pericardial friction rub is heard most frequently during expiration and increases when the patient is upright and leaning forward. Acute pericarditis, elevated white cell count, and a murmur heard over the left sternal border would not be the primary physical sign of the condition

A client complains of pain in the calves, thighs, and buttocks whenever he climbs more than a flight of stairs. This pain, however, is quickly relieved as soon as he sits down and rests. The nurse should suspect which of the following conditions in this client?

Peripheral arterial disease

A client reports pain in the legs that begins with walking but is relieved by rest. Which condition should the nurse assess the client for?

Peripheral vascular problems

Which action by a nurse demonstrates proper technique for assessment of chest expansion?

Place both hands on the posterior chest at T9, press thumbs together, and then ask client to take a deep breath

The nurse is assessing a client diagnoses with mitral stenosis. Which technique should the nurse use to listen to this condition?

Place the bell of the stethoscope over the apex with client on left side. Explanation: This mid-diastolic murmur is associated with an opening snap and has a low-pitched, rumbling quality. Heard best with the bell over the apex with the patient turned to the left. The carotid arteries are auscultated one at a time for bruits. The 2nd left intercostal space is the location to hear pulmonic valve conditions.

An older client is hospitalized with pneumonia. The nurse suspects the client is developing severe sepsis based on which assessment findings? (Select all that apply.)

Platelet count 90,000 Pulse 104 beats/minute PaCO2 30 mmHg

A 25-year-old male patient is brought to the emergency department by ambulance after being involved in a motor vehicle accident. You find that he has decreased breath sounds over the left lung fields. What might you suspect is the cause?

Pneumothorax

A nurse asks a supine client to raise his knee partially. The nurse ten places the thumbs on the knee while positioning the fingers deep in the bend of the knee. The nurse is palpating the pulse of which artery?

Popliteal

What pulse is located in the groove between the medial malleolus and the Achilles tendon?

Posterior tibial

A nurse monitors a client at risk for the onset of premature ventricular contractions. The nurse should monitor the client's cardiac rhythm for which characteristic feature?

Premature beats followed by compensatory pause

Upon assessment of a client's pulse, a nurse notices that the ampulitude of the pulse varies between beats. Which other findings should the nurse assess for in this client?

Presence of an S3

Upon assessment of a client's pulse, a nurse notices that the amplitude of the pulse varies between beats. Which other finding should the nurse assess for in this client?

Presence of an S3 Explanation: Changes in the amplitude (or strength) of a client's pulse from beat to beat is called pulsus alternans. This is usually seen in heart failure. The nurse should assess the client for the presence of an S3 and an S4, which indicate a noncompliant ventricle. Diminished heart sounds can be present in an obese client or with hypovolemia, shock, or decreased cardiac output. A pulse that changes with respirations is called a paradoxical pulse and seen in cardiac tamponade or obstructive lung disease. A split S2 does not change the amplitude of a client's pulse.

Which alteration in the pattern of the cardiac pulse should a nurse expect to find on exam of a client admitted w/ left ventricular failure?

Pulsus alternans

A nurse performs an ankle-brachial pressure index and obtains these results: R arm BP 140/90, L arm BP 150/90, R ankle 80, L ankle 85. Determine the right and left ABPI.

R .53, L .56 Divide highest brachia pressure by each arm number. 80/150 and 85/150

The nurse is preparing to conduct a peripheral vascular and lymphatic physical examination of a patient's legs. What instructions should the nurse provide the patient in order to prepare for this examination? (Select all that apply.)

Remove clothing below the waist. Lay on your back on a bed or examination table. Be prepared to lay on your stomach for a portion of the examination.

When percussing the anterior chest for tone, a nurse should anticipate what tone over the majority of the lung fields?

Resonance

Which area of the heart should be further assessed in the patient with an irregular pulse?

Right atrium

The nurse is auscultating heart tones. To auscultate the aortic area, the nurse would place the stethoscope where?

Right second intercostal space to apex of heart

When auscultating a client diagnosed with aortic stenosis, the nurse should place the stethoscope at what location on the client's chest?

Right sternal border, 2nd ICS Explanation: Aortic Stenosis is a midsystolic ejection murmur begins after S1, crescendos, and then decrescendos before S2. It radiates upward to the right second ICS and into the neck. It is soft to loud, with a medium pitch and harsh quality. It is associated with ejection click, split S2. It's best heard over the 2nd or 3rd right intercostal space.

During the auscultation of a patient's heart sounds, the nurse hears a fixed S2 split. What does this heart sound indicate to the nurse?

Right ventricular failure Explanation: Fixed splitting refers to wide splitting of the second heart sound that does not vary with respiration. It occurs in right ventricular failure. Wide splitting of the second heart sound occurs with a delayed closure of the pulmonic valve and is seen in pulmonic stenosis and right bundle branch block. Paradoxical or reversed splitting appears on expiration and disappears on inspiration. It is seen in left bundle branch block.

When describing the cardiac cycle to a group of students, the instructor correlates heart sounds with events of the cycle. Which heart sound would the instructor explain as being associated with systole?

S1

A client from a severe motor vehicle accident arrives in the emergency department. The nurse observes irregular respirations of varying depth and rate followed by periods of apnea. Which of the following would the nurse suspect?

Severe brain damage

A patient comes to the emergency department reporting a sudden onset of dyspnea. What finding is a manifestation of dyspnea?

Shortness of breath

While completing the cardiovascular system health history, a patient tells the nurse about using four pillows at night to sleep. The nurse will use this information to further assess which area?

Shortness of breath

During a physical exam, nurse notes client has slow, regular pulse. On cardiac monitor, QRS complexes are regular with normal P waves. The ventricular rate is found to be 54 beats/min. The nurse recognizes client may have abnormality in what part of conduction system?

Sinoatrial node

A nurse is reviewing the electrical conduction system of the heart in preparation for assessing a client with a conduction problem. The nurse should be aware that the electrical signal originates in which of the following locations?

Sinoatrial node Explanation: The sinoatrial node is often called the pacemaker of the heart because it generates impulses that are conducted through the heart. The impulse is conduced across the atria to the AV node, which then relays the impulse to the AV bundle or bundle of His. From here, the impulse travels down the right and left bundle branches and the Purkinje fibers in the myocardium of both ventricles.

A nurse ausculates the heart rate of a young male & notices that the rate speeds w/ inspiration & slows w/ exhalation. S1 & S2 are normal. The nurse recognizes this as what dysrhythmia?

Sinua arrhymia

The nurse notes that a client's heart rate increases with inspiration and slows down with expiration. How should the nurse document this finding?

Sinus arrhythmia

A student in the vascular surgery clinic is asked to perform a physical examination on a client with known peripheral vascular disease in the legs. Which of the following aspects are most important to note?

Size, symmetry, and skin colour

Which of the following occurs in respiratory distress?

Skin between the ribs moved inward with inspiration

increases cardiac workload and contributes to hypertension, plaque build-up, and blood clots.

Smoking

The nurse is discussing risk factors of an aneurysm. What should be included? Select all that apply.

Smoking Family history Hypertension

Goals, although not specific for peripheral vascular disease, focus on areas of risk. What are these areas of modifiable risk? Select all that apply.

Smoking Overweight Lack of exercise

A 52-year-old man is sceptical about the potentially harmful effect of his smoking on his heart, citing the fact that both his father and grandfather lived long lives despite being lifelong smokers. Which of the following facts would underlie the explanation that the nurse provides the client?

Smoking increases the heart's workload and contributes to atherosclerosis.

What teaching should be included to a client diagnosed with peripheral arterial disease? Select all that apply.

Stop smoking Low fat diet Increased activity Weight management

A high-pitched crowing sound from the upper airway results from tracheal or laryngeal spasm and is called what?

Stridor

A nurse is working with an older client who has decreased left ventricular compliance. The nurse understands that this condition will cause a decrease in the amount of blood pumped from the heart with each contraction, a measure known as which of the following?

Stroke volume Explanation: Stroke volume is the amount of blood pumped from the heart with each contraction. Cardiac output (CO) is the amount of blood pumped by the ventricles during a given period of time (usually 1 min) and is determined by the stroke volume (SV) multiplied by the heart rate (HR): SV × HR = CO. Neither systolic blood pressure nor heart rate measure the quantity indicated.

When auscultating a client's heart, the nurse hears both S3 and S4. What is this known as?

Summation gallop Explanation: Presence of both S3 and S4 is referred to as a "summation gallop." Atrial kick is the additional flow of blood from the atrium to the ventricles as the atrium contract. Ejection clicks are high-pitched sounds that occur at the moment of maximal opening of the aortic or pulmonary valves. They are heard just after the S1 sound. Diastolic clicks can be found in clients with mitral valve prolapse as the valve does not close properly.

A nurse is auscultating a client's heart sounds. What action should the nurse perform during this assessment?

Systematically listen to the entire precordium. Explanation: When auscultating heart sounds, the nurse would need to emphasize the need to cover the entire precordium, using a systematic approach moving the stethoscope from left to right across the entire heart area from the base to the apex or from the apex to the base. Both the diaphragm and bell are used. Inspection and palpation usually precede auscultation. It is not necessary to begin with breath sounds.

Which assessment finding would support the diagnosis of pericardial tamponade?

Systolic blood pressure decreases from 140 mm Hg to 127 mm Hg during inspiration.

indicates ventricular repolarization, when the ventricles return to a resting state.

T Wave

A group of nurses is reviewing several electrocardiograms (ECGs). The students demonstrate understanding of the waveforms when they identify which component as indicating ventricular repolarization?

T wave Explanation: The T wave indicates ventricular repolarization, when the ventricles return to a resting state. The P wave indicates atrial depolarization and conduction of the impulse throughout the atria. The QRS complex indicates ventricular depolarization with conduction of the impulse throughout the ventricles. The ST segment indicates the period between ventricular depolarization and the beginning of ventricular repolarization.

A nurse is reviewing a client's electrocardiogram (ECG). The nurse should identify which component as indicating ventricular repolarization?

T wave Explanation: The T wave indicates ventricular repolarization, when the ventricles return to a resting state. The P wave indicates atrial depolarization and conduction of the impulse throughout the atria. The QRS complex indicates ventricular depolarization with conduction of the impulse throughout the ventricles. The ST segment indicates the period between ventricular depolarization and the beginning of ventricular repolarization.

The staff educator from the hospital's respiratory unit is providing a public educational event. The educator is talking about health promotion activities for people with respiratory diseases or those who are at high risk for respiratory complications. What would the educator include in the presentation?

Teaching strategies to reduce complications of existing diagnoses

The sternal angle at the 2nd rib space is also known as what?

The aortic area

The nurse is assessing a client with a cardiac condition who complains of not sleeping well and of having to get up frequently at night to urinate. The nurse should recognize what implication of this statement?

The client may be experiencing symptoms of heart failure. Explanation: With heart failure, increased renal perfusion during periods of rest or recumbency may cause nocturia. This does not signal CAD, a conduction problem, or adequate compensation.

The nurse's auscultation of the client's heart sounds reveals the presence of a split S1. What conclusion should the nurse draw from this assessment finding?

The client's ventricles are not contracting simultaneously. Explanation: A split S1 occurs when the left and right ventricles contract at different times (asynchronous ventricular contraction). This finding is not associated with an incompetent aortic valve, left ventricular hypertrophy, or lack of synchronicity between the atria and ventricles.

creates the second heart sound, which is heard louder over the 2nd intercostal space right sternal border.

The closure of the aortic and pulmonic valves

In order for the nurse to assess jugular venous pressure (JVP), the client should be in which of the following positions?

The head of the bed raised 60 degrees Explanation: Jugular venous pressure reflects pressure in the right atrium or central venous pressure. With the head of the bed positioned at 60 degrees, the nurse can measure the jugular venous pressure because the "top" of the internal jugular vein is now visible, so the vertical distance from the sternal angle or right atrium can be measured. With the head of the bed positioned at 30 degrees, the jugular venous pressure cannot be measured because venous undulation is above the jaw and therefore, not visible. With the head of the bed positioned at 90 degrees, the veins are barely discernible above the clavicle, making measurement impossible. The jugular venous pressure cannot be measured with the client in a supine position because the head needs to be elevated slightly in order to bring the vein into view.

Where are the heart and great vessels located in the human body?

The heart and great vessels are located in the mediastinum between the lungs and above the diaphragm from the center to the left of the thorax.

A client complains of palpitations and a feeling of anxiety. Which of the following would be most appropriate for the nurse to keep in mind?

The heart is attempting to increase cardiac output. Explanation: Palpitations may occur with an abnormality of the heart's conduction system or during the heart's attempt to increase cardiac output by increasing the heart rate. Palpitations may cause the client to feel anxious.

Which of the following assessment findings would signal a pathophysiological finding to the nurse?

The intensity of the client's S1 varies between beats.

How should a nurse assess a client for pulse rate deficit?

The nurse should assess the pulse deficit by assessing the difference in the apical and radial pulse. Pulse deficit is the difference between the apical and peripheral/radial pulses. Differences in the amplitude or rate of the carotid pulse may indicate stenosis. A split S1 occurs when the left and right ventricles contract at different times (asynchronous contraction). Decrease in jugular venous pressure can occur with dehydration secondary to a decrease in total blood volume but does not cause a pulse deficit.

The nurse is assessing the jugular venous pressure (JVP) of a 72-year-old client with recent complaints of fatigue, shortness of breath, and swollen ankles. What cardiac phenomena are represented by the oscillations that the nurse observes in the client's internal jugular veins?

The pressures that exist within the client's right atrium Explanation: JVP is a visible manifestation of the varying pressures in the client's right atrium. It does not directly indicate contractility, valve function, preload, or afterload.

A new nurse on the telemetry unit is reviewing information about how to correctly read electrocardiograms. The nurse is expected to know that the PR interval represents what event?

The time from firing of the sinoatrial (SA) node to the beginning of depolarization in the ventricle Explanation: PR interval represents the time from the firing of the SA node to the beginning of ventricular depolarization (includes a slight pause at the AV junction).

The nurse is assessing the client's skin an ulcer is identified. What would indicate to the nurse it is a venous ulcer?

The ulcer is superficial and pale.

The nurse on the cardiac unit is caring for a patient who thinks he was having a myocardial infarction when he came to the emergency department. When reviewing laboratory data on this patient, the nurse notes that all tests are within normal limits except for the cholesterol and C-reactive protein, both of which are elevated outside the normal range. The nurse should be aware of what fact relating to elevated cholesterol and C-reactive protein?

They more than double the risk of cardiac disease. Explanation: The risk of a cardiovascular event more than doubles with an elevated cholesterol and C-reactive protein level.

While performing a routine check-up on an 81-year-old retired grain farmer in the vascular surgery clinic, the nurse notes that he has a history of chronic arterial insufficiency. Which of the following physical examination findings of the lower extremities would be expected with this disease?

Thin, shiny, atrophic skin

A nurse is working with a patient who has been confined to bed rest in the hospital for the past 2 weeks. Which areas of the body are most likely to develop ulcers due to arterial insufficiency? Select all that apply.

Tips of toes Toe webs Heels

The nurse is preparing discharge teaching for a patient diagnosed with a lymphatic disorder. What is one of the main teaching points the nurse should include?

To avoid sitting for long periods

On inspection of a client's legs, the nurse has found varicose veins. Which test should the nurse next perform to determine the competence of the saphenous vein valves?

Trendelenburg test

The radial pulse is palpated over the lateral flexor surface.

True

Which of the following veins drain into the superior vena cava? (Mark all that apply.)

Upper torso Head Upper extremities

A client might have an aortic regurgitation murmur. Which is the best position to accentuate the murmur?

Upright, but leaning forward Explanation: Leaning forward slightly in the upright position brings the aortic valve and the left ventricular outflow tract closer to the chest wall, so it will be easier to hear the soft diastolic decrescendo murmur of aortic insufficiency (regurgitation). The examiner can further hear this soft murmur by having the client hold his or her breath in exhalation.

Which statement describes the correct technique by a nurse when performing the ankle-brachial pressure index test?

Use a blood pressure cuff that is 20% wider than the diameter of the client's limb

Upon assessment, the nurse finds the left calf to be red and warm. The client states it only "aches". The nurse would suspect what?

Venous thromboembolism

When you enter the room of a hospitalized patient, you note that the patient is guarding her left leg, which is swollen and reddened. You should identify the signs and symptoms of what complication of hospitalization?

Venous thromboembolism

A client has been diagnosed with venous insufficiency. Which of the following findings should the nurse expect on interviewing this client?

Warm skin and brown pigmentation around the ankles

Which statement demonstrates correct technique by a nurse when using the Doppler device to locate the peripheral pulses?

Warming the gel will help to avoid vasoconstriction at the site

During ausculation of the heart, a nurse hears an extra heart sound immediately after S2 at the second left intercoastal space. What should the nurse do to further assess this finding?

Watch the client's respirations while listening for effect on the heart sound.

The nursing instructor is discussing assessment of the heart with students. A student states that he has a client with a rushing vibration in the precordium that the student could feel and that it was in the area of the pulmonic valve. What should the instructor explain that the student is feeling?

a thrill Thrills are vibrations detected on palpation. A palpable, rushing vibration (thrill) is caused from turbulent blood flow with incompetent valves, pulmonary hypertension, or septal defects. This vibration is usually in the location of the valve in which it is associated. A thrust or a heave is a forceful thrusting on the chest. This is not a normal finding.

refers to the degree of vascular resistance to ventricular contraction.

afterload

An adult client visits the clinic and tells the nurse that she feels chest pain and pain down her left arm. The nurse should refer the client to a physician for possible

angina. Explanation: Angina (cardiac chest pain) is usually described as a sensation of squeezing around the heart; a steady, severe pain; and a sense of pressure. It may radiate to the left shoulder and down the left arm or to the jaw.

The posterior tibial pulse can be palpated at the

ankle.

Understanding pressures in the left atrium, left ventricle, and aorta is fundamental to understanding heart sounds. Place the following in order of pressures and sounds through one cardiac cycle.

aortic pressure is greater than ventricular pressure mitral valve closes producing S1 the atrium is empty and pressures in the ventricles increase slightly S3 and S4 may be heard if pathologic ventricular compliance ventricular pressure increases and forces the opening of the aortic valve aortic valve closes producing S2

During a cardiac examination, the nurse can best hear the S1 heart sound by placing the stethoscope at the client's

apex of the heart. Explanation: S1 may be heard over the entire precordium but is heard best at the apex (left MCL, fifth ICS).

The apex of each lung is located at the

area slightly above the clavicle.

While assessing the peripheral vascular system of an adult client, the nurse detects cold clammy skin and loss of hair on the client's legs. The nurse suspects that the client may be experiencing

arterial insufficiency.

The nurse is preparing to auscultate the posterior thorax of an adult female client. The nurse should

ask the client to breathe deeply through her mouth

During assessment, the nurse notes an irregular rhythm. What should the nurse do next?

assess for a pulse deficit

The nurse is having difficulty locating a client's point of maximum impulse. What should the nurse do to facilitate this assessment?

assist the client into a left lateral decubitus position Explanation: If unable to identify the apical impulse with the client supine, assist the client to roll partly onto the left side or the left lateral decubitus position. The nurse was unable to locate the client's point of maximum impulse in the supine position. Sitting with the legs dangling and the high-Fowler's position are not positions that will help locate the point of maximum impulse.

While assessing an older adult client, the nurse detects a bruit over the carotid artery. The nurse should explain to the client that a bruit is

associated with occlusive arterial disease. Explanation: A bruit, a blowing or swishing sound caused by turbulent blood flow through a narrowed vessel is indicative of occlusive arterial disease.

The semilunar valves are located

at the exit of each ventricle at the beginning of the great vessels. Explanation: The semilunar valves are located at the exit of each ventricle at the beginning of the great vessels.

The bicuspid, or mitral, valve is located

between the left atrium and left ventricle

The largest arteries of the upper extremities are the

brachial arteries

After palpating the radial pulse of an adult client, the nurse suspects arterial insufficiency. The nurse should next assess the client's

brachial pulse.

A nurse auscultates a client's heart sounds and obtains a rate of 56 beats per minute. How should this rate be documented by the nurse?

bradycardia The proper documentation of this rate is bradycardia, a rate less than 60 beats per minute. The normal adult heart rate is 60 to 100 beats per minute. Tachycardia is a heart rate above 100 beats per minute. This heart rate is decreased, but this is not a proper documentation term.

The nurse hears high-pitched swooshing sounds over the carotid artery on the right side. What is this sound indicative of?

bruits- Distinguishing a murmur from a bruit can be challenging. Murmurs originate in the heart or great vessels and are usually louder over the upper precordium and quieter near the neck. Bruits are higher pitched, more superficial, and heard only over the arteries. A gallop is a generic term for an additional heart sounds heard besides the normal S1 and S2 sound.

Mitral valve sounds are usually heard best at and around the

cardiac apex

A student is asked to define the continuous rhythmic movement of blood during contraction and relaxation of the heart. This best describes which of the following?

cardiac cycle

While palpating the apex, left sternal border, and base in an adult client, the nurse detects a thrill. The nurse should further assess the client for

cardiac murmur. Explanation: A thrill or a pulsation is usually associated with a grade IV or higher murmur.

A nurse provides prevention strategies to a group of clients who are identified as at risk for hypertension. Which strategies should the nurse include? Select all that apply.

choose foods like bananas and sweet potatoes walk briskly 30 mins a day low salt

The lining of the trachea and bronchi, which serves to remove dust, foreign bodies, and bacteria, is termed the

cilia

A nurse is unable to palpate the apical impulse on an older client. Which assessment data in the client's history should the nurse recognize as the reason for this finding?

client has an increased chest diameter The apical impulse may not be palpable in clients with increased anteroposterior diameters. Irregular heart rate should not interfere with the ability to palpate an apical impulse. Respiratory rate does not impact the apical impulse. Heart enlargement would displace the apical impulse but not cause it to be nonpalpable.

During the lung assessment for a client with pneumonia, the nurse auscultates low-pitched, bubbling, moist sounds that persist from early inspiration to early expiration. How should the nurse document these sounds?

coarse crackles

The nurse is planning to perform the Trendelenburg test on an adult client. The nurse should explain to the client that this test is used to determine the

competence of the saphenous vein valves.

The P-wave phase of an electrocardiogram (ECG) represents

conduction of the impulse throughout the atria. Explanation: The P wave indicates atrial depolarization; conduction of the impulse throughout the atria.

A 68-year-old mechanic presents to the emergency room for shortness of breath. The examiner is concerned about a cardiac cause and measures the client's jugular venous pressure (JVP). It is elevated. Which of the following conditions is a potential cause of elevated JVP?

constrictive pericarditis

During an interview with the nurse, a client complains of a fatigue that seems to get worse in the evening. Which of the following causes of fatigue would explain this pattern?

decreased cardiac output-Fatigue may result from compromised cardiac output. Fatigue related to decreased cardiac output is worse in the evening or as the day progresses, whereas fatigue seen with depression is ongoing throughout the day. Severe muscular exertion and an upper respiratory infection may be associated with fatigue, but not the pattern mentioned in the scenario.

a blood pressure falls to its lowest in

diastole

Which characteristic of the first heart sound would the nurse expect to hear in a client with mitral insufficiency?

diminished

Which characteristic of the apical pulse should a nurse expect to find in the client diagnosed with left ventricular hypertrophy?

displaced The nurse should expect to find a displaced apical pulse for a client with left ventricular hypertrophy. In ventricular hypertrophy, the apical pulse may be larger than 1 to 2 cm, displaced, more forceful, or of longer duration. Bounding apical pulse is not a characteristic of ventricular hypertrophy.

The nurse is conducting a health history with a female client who reports upper back and jaw pain. In order to assess the client's risk for a cardiac event, which question should the nurse ask first?

do you have any pain or discomfort in your chest? The first question the nurse asks should be broad as this will encourage the client to share more detail regarding the source of the pain. Chest pain is one of the most serious and important symptoms often signaling coronary artery disease, potentially leading to myocardial infarction. All of the other options are more specific; these questions should only be asked when the nurse needs to narrow the focus of the cardiovascular examination.

The nurse is caring for a client who has an elevated cholesterol level. To reduce the mean total blood cholesterol and LDL cholesterol levels, what would be important to teach this client?

eat low fat. low cholesterol meals

The nurse is preparing to palpate the epitrochlear lymph nodes of an adult male client. The nurse should instruct the client to

flex his elbow about 90 degrees.

The nurse is caring for a client who is employed as a typist and has a family history of peripheral vascular disease. The nurse should instruct the client to reduce her risk factors by

getting regular exercise.

The nurse hears a quiet murmur immediately after placing the stethoscope on the chest. Documentation of grading for this murmur would include

grade 2

During the health history interview with a 40-year-old man, the nurse uses the genogram to specifically assess for major family risk for cardiovascular disease by asking about which of the following?

heart attacks in his father or siblings Risk of developing heart disease is increased if one or more immediate family members (parents or siblings) have had an MI, hypertension, or high cholesterol.

An adult client tells the nurse that his father died of a massive coronary attack at the age of 65. The nurse should explain to the client that one of the risk factors for coronary heart disease is

high serum level of low-density lipoproteins. Explanation: Dyslipidemia presents the greatest risk for the developing coronary artery disease. Elevated cholesterol levels have been linked to the development of atherosclerosis.

Pericardial friction rubs can be differentiated from pleural friction rubs by having the client ________

hold their breath

The nurse assesses edema in a newly admitted patient. Further evaluation is based on the fact that the nurse knows edema is caused by (Select all that apply.)

increased capillary blood pressure increased capillary membrane permeability low plasma protein levels blockage of lymphatic drainage

Which of the following would the nurse suspect when a client with a cardiac condition complains of not sleeping well and having to get up frequently at night to urinate?

increased urination at rest may indicate heart failure

The nurse assesses a hospitalized adult client and observes that the client's jugular veins are fully extended. The nurse contacts the client's physician because the client's signs are indicative of

increased venous pressure The level of the jugular venous pressure reflects right atrial (central venous) pressure and, usually, right ventricular diastolic filling pressure. Right-sided heart failure raises pressure and volume, thus raising jugular venous pressure.

What nursing diagnosis would be most appropriate for a client admitted with heart failure?

ineffective tissue perfusion Heart failure can cause ineffective tissue perfusion which can lead to fatigue, pain and activity intolerance. Impaired gas exchange would be more appropriate for respiratory disorders

A client is admitted to the health care facility with reports of chest pain, elevated blood pressure, and shortness of breath with activity. The nurse palpates the carotid arteries as 1+ bilaterally and a weak radial pulse. A Grade 3 systolic murmur is auscultated. Which nursing diagnosis can the nurse confirm based on this data?

ineffective tissue perfusion' The nurse assesses a decrease in the carotid pulses (1+ is considered weak) and a weak radial pulse is present. The client also has a murmur. These findings allow the nurse to confirm the diagnosis of Ineffective Tissue Perfusion. There are not enough criteria to confirm the diagnosis of Impaired Breathing Pattern, Activity Intolerance, or Ineffective Health Maintenance.

The nurse assesses a decrease in the carotid pulses (1+ is considered weak) and a weak radial pulse is present. The client also has a murmur. These findings allow the nurse to confirm the diagnosis of

ineffective tissue perfusion.

A nurse cares for a client who suffered a myocardial infarction 2 days ago. A high-pitched, scratchy, scraping sound is heard that increases with exhalation and when the client leans forward. The nurse recognizes this sound as a result of what process occurring within the pericardium?

inflammation of pericardial sac A high pitched, scratchy, scraping sound that increases with exhalation and when the client leans forward is called a pericardial friction rub. This is caused by inflammation of the pericardial sac. Increased pressure within the ventricles may cause a decrease in cardiac output. Inability of the atria to contract can be caused by any problem that causes the sinoatrial node not to fire. An incompetent mitral valve would cause a systolic murmur.

A nurse cares for a client who suffered a myocardial infarction two (2) days ago. A high pitched, scratchy, scraping sound is heard that increase with exhalation and when the client leans forward. The nurse recognizes this sound as a result of what process occurring within the pericardium?

inflammation of the pericardial sac A high pitched, scratchy, scraping sound is heard that increase with exhalation and when the client leans forward is called a pericardial friction rub. This is caused by inflammation of the pericardial sac. Increased pressure within the ventricles may cause a decrease in cardiac output. Inability of the atria to contract can be caused by any problem that causes the sinoatrial node not to fire. An incompetent mitral valve would cause a systolic murmur.

The nurse is assessing the peripheral vascular system of an older adult client. The client tells the nurse that her legs "seem cold all the time and sometimes feel tingly." The nurse suspects that the client may be experiencing

intermittent claudication.

Across the lifespan, a nurse knows that the female heart

is usually smaller and weighs less than the male heart

The popliteal artery can be palpated at the

knee.

In order to palpate an apical pulse when performing a cardiac assessment, where should the nurse place the fingers?

left midclavicular line at the 5th intercostal space The apical pulse is the point of maximal impulse and is located in the fifth intercostal space at the left midclavicular line when the client is placed in a sitting position. The apical impulse is palpated in the mitral area and therefore cannot be palpated at the left midclavicular line at the third intercostal space, at right of the midclavicular line at the third intercostal space and at right of the midclavicular line at the fifth intercostal space.

A client visits the clinic and tells the nurse that she had a mastectomy 2 years ago. The nurse should assess the client for

lymphedema.

While assessing the inguinal lymph nodes in an older adult client, the nurse detects that the lymph nodes are approximately 3 cm in diameter, nontender, and fixed. The nurse should refer the client to a physician because these findings are generally associated with

malignancy.

While assessing an adult client, the nurse detects opening snaps early in diastole during auscultation of the heart. The nurse should refer the client to a physician because this is usually indicative of

mitral valve stenosis. Explanation: Opening snaps occur early in diastole and indicate mitral valve stenosis.

In auscultating a client's heart sounds, a nurse hears a swooshing sound over the pre cordium. The nurse recognizes this sound as which of the following?

murmur Blood normally flows silently through the heart. There are conditions, however, that can create turbulent blood flow in which a swooshing or blowing sound may be auscultated over the pre cordium; this sound is known as a murmur. S1, the first heart sound, sounds like "lub," and S2, the second heart sound, sounds like "dubb." Ventricular gallop is a name for the third heart sound, S3, which is not a swooshing sound over the pre cordium.

A group of students is reviewing the structures of the heart, noting that the thickest layer of the heart is made up of contractile muscle cells. The students are correct in identifying it as which of the following?

myocardium

A nurse is working with a client who recently suffered a heart attack. As a result, the client has experienced the death of the muscle tissues that make up the thickest layer of the heart. This layer of muscle is known as which of the following?

myocardium The myocardium is the thickest layer of the heart and is made up of contractile cardiac muscle cells. The pericardium is a tough, inextensible, loose-fitting, fibroserous sac that attaches to the great vessels and surrounds the heart. A serous membrane lining, the parietal pericardium, secretes a small amount of pericardial fluid that allows for smooth, friction-free movement of the heart. This same type of serous membrane covers the outer surface of the heart and is known as the epicardium. The endocardium is a thin layer of endothelial tissue that forms the innermost layer of the heart and is continuous with the endothelial lining of blood vessels.

Auscultation of a 23-year-old client's lungs reveals an audible wheeze. What pathological phenomenon underlies wheezing?

narrowing or partial obstruction of an airway passage

Which of the following would the nurse consider to be an urgent situation?

new onset chest pain

A nurse performs an initial health history on a client admitted for new onset of chest pain. Which data is considered subjective for the cardiovascular system?

no current medications or treatment Subjective data is data collected from the client. No current medications or treatments is information the nurse obtained from the client. Apical heart rate 70 beats per minute, no edema of extremities noted, and apical impulse palpated at 5 intercostal space on left are examples of objective data collected by the nurse upon physical examination.

A client complains of difficulty sleeping, stating he has to sit up with the help of several pillows and cannot breathe when lying flat. This client has a condition known as what?

orthopnea

A 58-year-old teacher presents with breathlessness with activity. The client has no chronic conditions and does not take any medications, herbs, or supplements. Which of the following symptoms is appropriate to ask about in the cardiovascular review of systems?

orthopnea- Orthopnea, which is dyspnea that occurs when lying down and improves when sitting up, is part of the cardiovascular review of systems and, if positive, may indicate congestive heart failure.

A nursing student is reviewing the electrical conduction of the heart. The student is correct in identifying the sinoatrial node of the heart as which of the following?

pacemaker

The nurse manager on a cardiac unit should immediately intervenen when observing which staff nurse's assessment technique?

palpating carotid pulses simultaneously Carotid pulse palpation should be conducted by feeling one side at a time; otherwise the client my become dizzy or lightheaded. All other assessment techniques are correct.

The thin double-layered serous membrane that lines the chest cavity is termed

parietal pleura

While auscultating the heart at the third intercostal space, left sternal border, the nurse notes a high-pitched, scratchy sound that increases with exhalation with the client leaning forward. The nurse would document which of the following?

pericardial friction rub

is best heard in the third intercostal space at the left sternal border and is associated with a high-pitched, scratchy sound caused by inflammation of the pericardial sac.

pericardial friction rub

A high pitched, scratchy, scraping sound is heard that increase with exhalation and when the client leans forward is called a pericardial friction rub. This is caused by inflammation of the

pericardial sac

A client reports sharp and stabbing chest pain that worsens with deep breathing and coughing. A cardiac cause to this pain is ruled out. The description of the pain is consistent with what respiratory condition?

pleurisy

After assessing pitting edema below the knee in a patient, the nurse would suspect that which vein may be occluded?

popliteaL

The anterior chest area that overlies the heart and great vessels is called the

precordium. Explanation: The anterior chest area that overlies the heart and great vessels is called the precordium.

A patient with prehypertension is in the clinic for counseling. All lifestyle modifications are important in preventing or managing hypertension. Which modification would be the best to implement first for this patient?

regular exercise for at least 30 minutes a day Explanation: Regular exercise provides many benefits, including lowering the risk of hypertension.

A client has engorged jugular veins. What should this finding suggest to the nurse?

right atrial pressure Jugular venous pressure (JVP) reflects right atrial pressure. Engorged jugular veins are seen in right or left heart failure, pulmonary hypertension, tricuspid stenosis, and pericardial compression or tamponade. The jugular veins are not used to estimate the integrity of the aorta, patency of carotid arteries, or the closure of the tricuspid valves.

The nurse performs an admission assessment on an adult client admitted through the ED with a myocardial infarction. The nurse charts "Swooshing sound heard over right carotid artery." How should this documentation be corrected?

right carotid bruit auscultated-Bruits are swooshing sounds similar to the sound of the blood pressure. They result from turbulent blood flow related to atherosclerosis. A bruit is audible when the artery is partially obstructed. With complete obstruction, no bruit is audible, because no blood gets through. Distinguishing a murmur from a bruit can be challenging. Murmurs originate in the heart or great vessels and are usually louder over the upper precordium and quieter near the neck. Bruits are higher pitched, more superficial, and heard only over the arteries. Split sounds are not heard over arteries.

The nurse begins auscultating a client's heart sounds at the 2nd intercostal space right sternal border. Which location should the nurse assess next?

second intercostal space left sternal border Since the nurse started at the base of the heart, the next location to assess would be the 2nd intercostal space left sternal border. The 3rd left intercostal space would be assessed next and followed by the 4th intercostal space. The 5th left intercostal space midclavicular line would be assessed last.

As the heart rate increases, the length of diastole is

shortened

A client comes to the emergency department reporting a sudden onset of dyspnea. What finding is a manifestation of dyspnea?

shortness of breath Clients with heart failure may be short of breath from fluid accumulation in the pulmonary bed. Onset may be sudden with acute or chronic pulmonary edema. It is important to assess how much activity brings on dyspnea, such as rest, walking on a flat surface, or climbing. The other options listed are distracters to the question.

While completing the cardiovascular system health history, a client reports difficulty falling asleep unless she is in an upright position. Which of the following potential problems should the nurse further investigate?

shortness of breath Shortness of breath, also called orthopnea, is dyspnea that occurs while the client is lying flat and improves when the client sits up. The client would not experience relief from chest pain, palpitations or edema by sitting upright. For this reason, these options are incorrect.

The nurse notes that a client's heart rate speeds up with inspiration and slows down with expiration. What should the nurse suspect this client is demonstrating?

sinus arrhythmia Explanation: In a sinus arrhythmia the heart varies cyclically, usually speeding up with inspiration and slowing down with expiration. In atrial fibrillation the ventricular rhythm is totally irregular, although short runs of the irregular ventricular rhythm may seem regular. In premature atrial contractions a beat of atrial origin comes earlier than the next expected normal beat. A pause follows, and then the rhythm resumes. In premature ventricular contractions a beat of ventricular origin comes earlier than the next expected normal beat. A pause follows, and the rhythm resumes.

A nurse auscultates the heart rate of a young male and notices that the rate speeds with inspiration and slows with exhalation. S1 and S2 are normal. The nurse recognizes this as what dysrhythmia?

sinus arrhythmia - A heart rate that speeds with inspiration and slows with exhalation is termed Sinus Arrhythmia. This is often a normal rhythm in young children and well conditioned athletes. Premature Ventricular Contractions and Premature Atrial Contractions occur earlier than expected. Atrial Fibrillation causes the ventricles to beat irregularly.

Across the lifespan, a nurse knows that the female heart is normally________than the male heart

smaller

A 52-year-old man is skeptical about the potentially harmful effect of his smoking on his heart, citing the fact that both his father and grandfather lived long lives despite being lifelong smokers. Which of the following facts would underlie the explanation that the nurse provides the client?

smoking increases hearts workload and leads to atherosclerosis Smoking increases cardiac workload and contributes to hypertension, plaque build-up, and blood clots. It does not directly affect contractility or cardiac conduction, and it is not a component of metabolic syndrome.

The nurse documents vesicular lung sounds upon auscultation. The nurse heard what type of sound?

sound heard throughout inspiration and two thirds of expiration

A patient has had consecutive blood pressure readings in the 140s/90s for the last week of evaluation. This would be classified as

stage 1 hypertension

Blood pressure in the arterial system varies during the cardiac cycle, peaking in

systole

What finding upon assessment would indicate the client is experiencing shock?

systolic blood pressure of 50

The nurse is auscultating the heart sounds of an adult client. To auscultate Erb point, the nurse should place the stethoscope at the

third to fifth intercostal space at the left sternal border.

The nurse auscultates the apical pulse and then palpates the PMI (point of maximal impulse). To best palpate the PMI, the nurse places two fingers at the left border of the heart in the 5th intercostal space. (t/f)

true

A client has a brownish discoloration of the skin of both lower legs. What should the nurse suspect is occurring with this client?

venous insufficiency

During a physical examination, the nurse detects warm skin and brown pigmentation around an adult client's ankles. The nurse suspects that the client may be experiencing

venous insufficiency.Cigarette smoking Previous use of hormones High-fat diet

After teaching a group of students about blood flow through the heart, the instructor determines that the teaching was successful when the students state that after being received by the atria, the blood goes to which of the following?

ventricles

is the carotid pulse palpable

yes

The nurse is preparing to conduct a physical examination of a client's cardiovascular system. Which of the following instructions should the nurse provide the client in preparation for this examination? (Select all that apply.)

• "Wear the gown with the opening in the front." • "You will be laying on your left side for part of the examination." • "You will be sitting and leaning forward for part of the examination." Explanation: When preparing the client for a physical examination of the cardiovascular system, the nurse should instruct the client to put on the examination gown with the opening in the front for access to fully auscultate the chest wall. The nurse should tell the client he will be both left lying and sitting, leaning forward for part of the examination. These positions help bring the ventricular apex and left ventricular outflow tract closer to the chest wall, enhancing detection of the point of maximal impulse (PMI) and aortic insufficiency. The client will be supine, side lying or sitting during the examination but will not be standing. The nurse will conduct the examination from the patient's right side, not the left.

A 25-year-old client presents to the health care clinic for a routine physical. Which lifestyle practices in the client's history are most important for the nurse to provide teaching in assisting the client to reduce the risk for the development of coronary heart disease? Select all that apply.

• *Smokes 5 cigarettes daily for past 5 years* • *Family history of heart attack before age 50 years* • *High stress job as a financial analyst* -Adds salt to foods regularly -Exercise 30 mins 5X a week -Ten pounds over ideal body weight Risk factors for the development of coronary heart disease for this client include tobacco use, family history of heart attack before age 50 years, and a high-stress job. Exercise would tend to lower the client's risk for heart disease, not increase it. Although the client is 10 pounds over his ideal body weight, there is no evidence that he is overweight or obese. Also, although the client adds salt to his food regularly, there is no indication that he has high blood pressure. P443

Which of the following interventions can the nurse use to help a client modify risk factors associated with with hypertension? (Select all that apply).

• Ask the client to keep a "typical day" record of daily food intake • Ask the client to keep an activity log • Provide information about aids to promote smoking cessation Explanation: Interventions that would modify risk factors associated with hypertension include changing dietary intake, increasing physical activity and promoting smoking cessation. Asking the client to keep a "typical day" record of daily intake can help the nurse identify good diet choices and areas for improvement. Asking the client to keep an activity log offers the nurse objective information about the client's activity level in order to provide the client with strategies for increasing physical activity to the recommended 30 minutes per day. Providing information regarding smoking cessation is a step toward moving the client to the contemplation stage in the Stages of Change model. Moderate alcohol consumption per day of two drinks for men and one drink per day for women is considered acceptable for clients diagnosed with hypertension. Dietary intake of more than 3500 mg of potassium daily is encouraged for clients with hypertension.

Nursing students are learning how to objectively assess venous pulses. How would the nursing instructor teach the students to optimally position the patient for this procedure?

• Emphasize the shadows of the pulsations with lighting • Place the head of the bed 30° to 45° • Have the patient turn the head away from the side being examined

A nurse receives a client from the cardiac catheterization laboratory. The client is on bed rest and has a weight on the puncture site of the right groin. What must the nurse assess for frequently? Select all that apply.

• Hematoma at the puncture site • Bleeding at the puncture site • Increased blood pressure, which could cause excess bleeding or hematoma Explanation: Left-sided heart catheterization involves placing a catheter through the femoral artery to the coronary arteries where dye is used for visualization. Following the procedure, the client is on bed rest and the puncture site and distal circulation must be monitored frequently. Nursing staff also monitor blood pressure and cardiac rhythm.

A 25 year old client presents to the health care clinic for a routine physical. Which lifestyle practices in the client's history should the nurse provide teaching to assist the client to reduce the risk for the development of coronary heart disease?

• Smokes 5 cigarettes daily for past 5 years • Family history of heart attack before age 50 years • High stress job as a financial analyst

To increase the proportion of adults who engage in vigorous physical activity that promotes the development and maintenance of cardiorespiratory fitness, what would the nurse teach a patient

• The importance of physical activity • To exercise vigorously 3 or more days a week • To exercise for 20 or more minutes

A nurse is preparing a class for a local community group on coronary heart disease. Which of the following recommendations would the nurse include as appropriate for reducing a person's risk? Select all that apply.

• Use relaxation techniques to manage stress. • Walk for at least 30 minutes/day. • Eat foods low in sodium. Explanation: Measures to reduce the risk of CHD include eating 3½ ounces equivalent of cocoa such as dark chocolate each day to help lower blood pressure; eating foods low in saturated fats, trans fatty acids, cholesterol, and sodium; participating in an active exercise program such as walking at least 30 minutes per day; limiting alcohol intake to 2 drinks per day for men and 1 drink per day for women; managing stress by reducing personal stress as much as possible, trying muscle relaxation and deep breathing.

A client complains of chest pain. The nurse understands that chest pain can have causes other than cardiac pain, thus follows up with the client regarding the timing and quality of this pain. Which of the following would indicate cardiac pain as opposed to other types? Select all that apply.

• Worsens with activity • May occur at any time • Radiates to left shoulder and down the left arm • Has a squeezing sensation around the heart Explanation: Chest pain can be cardiac, pulmonary, muscular, or gastrointestinal in origin. Angina (cardiac chest pain) is usually described as a sensation of squeezing around the heart; a steady, severe pain; and a sense of pressure. It may radiate to the left shoulder and down the left arm or to the jaw. Cardiac pain may occur anytime, is not relieved with antacids, and worsens with activity. Gastrointestinal pain may occur after meals and is relieved with antacids.


Conjuntos de estudio relacionados

Pathophysiology Week 12- Inflammatory Bowel Disease

View Set

Chapter 16 Michigan Life, Accident, and Health Insurance Law

View Set

What was the significance of Greece as a classical empire?

View Set

Exam 1 - Death and Dying Chapters 1, 2, 3, & 4

View Set